Shift by one

Algebra Level 5

Find the sum of all complex values of the parameter A A , such that that polynomial f A ( x ) = x 6 + 6 x 5 + 5 x 4 30 x 3 74 x 2 72 x + A ( x 4 + 5 x 3 + 9 x 2 + 8 x ) \begin{aligned} f_A(x)=& x^6+6x^5+5x^4-30x^3-74x^2-72x \\ & +A\left(x^4+5x^3+9x^2+8x\right) \\ \end{aligned}

can be expressed as g ( h ( x ) ) g(h(x) ) , where g ( x ) g(x) and h ( x ) h(x) are non-linear polynomials with complex coefficients.


The answer is 22.

This section requires Javascript.
You are seeing this because something didn't load right. We suggest you, (a) try refreshing the page, (b) enabling javascript if it is disabled on your browser and, finally, (c) loading the non-javascript version of this page . We're sorry about the hassle.

2 solutions

C Lim
Sep 16, 2013

We say that a polynomial is nice if it can be written as g ( h ( x ) ) g(h(x)) , where g ( x ) g(x) and h ( x ) h(x) are non-linear polynomials.

Step 1 : If f ( x ) f(x) is nice, then so is f ( x c ) f(x-c) for any constant c c .

Proof : If f ( x ) = g ( h ( x ) ) f(x) = g(h(x)) , we have f ( x c ) = g ( h ( x c ) ) f(x-c) = g(h(x-c)) so it suffices to replace g ( x ) , h ( x ) g(x), h(x) with g ( x ) , h ( x c ) g(x), h(x-c) respectively. (QED)

Hence, we may get rid of the x 5 x^5 term in f A ( x ) f_A(x) by substituting x x with x 1 x-1 to obtain:

f A ( x 1 ) = ( x 6 10 x 4 10 x 3 + x 2 10 x + 28 ) + A ( x 4 + x 3 + x 3 ) \begin{aligned} f_A(x-1) &= (x^6 - 10x^4 - 10x^3 + x^2 - 10x + 28)\\ &+ A(x^4 + x^3 + x - 3) \end{aligned}

Step 2 : If f ( x ) f(x) is nice and f ( x ) = g ( h ( x ) ) f(x) = g(h(x)) for non-linear g ( x ) , h ( x ) g(x), h(x) , we may assume with no loss of generality that h ( 0 ) = 0 h(0) = 0 and h ( x ) h(x) is monic.

Proof : For any constants α , β ( α 0 ) \alpha, \beta (\alpha \ne 0) , replace g ( x ) , h ( x ) g(x), h(x) by g ( ( x β ) / α ) , α h ( x ) + β g((x-\beta)/\alpha), \alpha h(x)+\beta respectively. We can choose α , β \alpha, \beta such that h ( x ) h(x) is monic and has no constant term. (QED)

Step 3 : For f A ( x 1 ) f_A(x-1) above, if deg ( g ) = 3 \deg(g) = 3 and deg ( h ) = 2 \deg(h) = 2 , then A = 10 A = 10 is the only solution.

Proof : By step 2, write h ( x ) = x 2 + b x h(x) = x^2 + bx . Since the coefficient of x 5 x^5 is zero, we have b = 0 b=0 as well. Hence, h ( x ) = x 2 h(x) = x^2 and all the odd terms of f A ( x 1 ) f_A(x-1) vanish, i.e. A = 10 A=10 . Conversely, for A = 10 A=10 , all odd terms vanish so f A ( x 1 ) f_A(x-1) is a polynomial in x 2 x^2 . (QED)

Step 4 : For f A ( x 1 ) f_A(x-1) above, if deg ( g ) = 2 \deg(g) = 2 and deg ( h ) = 3 \deg(h) = 3 , then A = 12 A = 12 is the only solution.

Proof : Write h ( x ) = x 3 + b x 2 + c x h(x) = x^3 + bx^2 + cx by step 2. Once again, by step 1, b = 0 b=0 . So we have:

f A ( x 1 ) = p ( x 3 + c x ) 2 + q ( x 3 + c x ) + r f_A(x-1) = p(x^3 + cx)^2 + q(x^3 + cx) + r

for some quadratic g ( x ) = p x 2 + q x + r g(x) = px^2 + qx + r . Comparing the coefficients, we get:

p = 1 , 2 p c = A 10 , p c 2 = 1 , q = A 10 , q c = A 10. p=1,\ 2pc = A-10,\ pc^2 = 1,\ q=A-10,\ qc = A-10.

The last two equations give us q = q c q=qc . If q = 0 q=0 , then A = 10 A=10 as in step 3. Otherwise, c = 1 c=1 and we have A = 12 A=12 , in which case we can write f A ( x 1 ) f_A(x-1) as:

x 6 + 2 x 4 + 2 x 3 + x 2 + 2 x + 8 = ( x 3 + x ) 2 + 2 ( x 3 + x ) + 8. \begin{aligned} & x^6 + 2x^4 + 2x^3 + x^2 + 2x + 8 \\ = & (x^3 + x)^2 + 2(x^3 + x) + 8. \end{aligned}

Moderator note:

Great job!

respect! :')

Aditya Pappula - 7 years, 8 months ago
Mark Hennings
Sep 17, 2013

Since g ( h ( 0 ) ) = 0 g(h(0))=0 , we note that, replacing g ( x ) g(x) and h ( x ) h(x) by g ^ ( x ) = g ( x + h ( 0 ) ) \hat{g}(x) = g(x+h(0)) and h ^ ( x ) = h ( x ) h ( 0 ) \hat{h}(x) = h(x) - h(0) , we may assume without loss of generality that g ( 0 ) = h ( 0 ) = 0 g(0) = h(0) = 0 . Since g ( h ( x ) ) g(h(x)) is sextic and neither g ( x ) g(x) nor h ( x ) h(x) is linear, we deduce that one of g ( x ) g(x) and h ( x ) h(x) is quadratic, and the other is cubic. We may also assume that g ( x ) g(x) is monic, since any leading coefficient for g ( x ) g(x) can be incorporated into h ( x ) h(x) .

  1. If g ( x ) g(x) is quadratic, write g ( x ) = x 2 + a x g(x) = x^2 + ax , h ( x ) = α x 3 + β x 2 + γ x h(x) = \alpha x^3 + \beta x^2 + \gamma x . Multiplying out g ( h ( x ) ) g(h(x)) and matching coefficients, we deduce that α 2 = 1 2 α β = 6 β 2 + 2 α γ = A + 5 2 β γ + a α = 5 A 30 γ 2 + a β = 9 A 74 a γ = 8 A 72 \begin{array}{rclrcl} \alpha^2 & = & 1 & 2\alpha\beta &=& 6 \\ \beta^2 + 2\alpha\gamma &=& A+5 & 2\beta\gamma + a\alpha &=& 5A-30 \\ \gamma^2 + a\beta &=& 9A - 74 & a\gamma & = & 8A-72 \end{array} These equations can be solved to obtain A = 12 A=12 and α 2 = 1 β = 3 α γ = 4 α a = 6 α \alpha^2 = 1 \qquad \beta =3\alpha \qquad \gamma = 4\alpha \qquad a = 6\alpha

  2. If g ( x ) g(x) is cubic, write g ( x ) = x 3 + a x 2 + b x g(x) = x^3 + ax^2 + bx , h ( x ) = α x 2 + β x h(x) = \alpha x^2 + \beta x . Multiplying out g ( h ( x ) ) g(h(x)) and matching coefficients, we deduce that α 3 = 1 3 α 2 β = 6 3 α β 2 + a α 2 = A + 5 β 3 + 2 a α β = 5 A 30 a β 2 + b α = 9 A 74 b β = 8 A 72 \begin{array}{rclrcl} \alpha^3 & = & 1 & 3\alpha^2\beta & = & 6 \\ 3\alpha\beta^2 + a\alpha^2 & = & A+5 & \beta^3 + 2a\alpha\beta & = & 5A-30 \\ a\beta^2 + b\alpha & = & 9A-74 & b\beta & = & 8A-72 \end{array} These equations can be solved to obtain A = 10 A=10 and α 3 = 1 β = 2 α a = 3 α b = 4 α 2 \alpha^3 = 1 \qquad \beta = 2\alpha \qquad a = 3\alpha \qquad b = 4\alpha^2

Thus the only possible values of A A are 12 12 and 10 10 , and 12 + 10 = 22 12+10 = 22 .

Moderator note:

Great job!

... it was my idea too ... but I obtained one more solution in case 1, A=9, and I didn't check all the equations! Terrible mistake :(

Virgilius Teodorescu - 7 years, 8 months ago

0 pending reports

×

Problem Loading...

Note Loading...

Set Loading...